2x = 4y + 8, x - 2y = 4
System of Equations

Answers

Answer 1

Answer:

Step-by-step explanation:

2x=4y+8

divide by 2

x=2y+4

and x-2y=4

or x=2y+4

so both the equations represent the same line.


Related Questions

Which of the following is equal to 5^1/3?
A. ^3√5 B. 5^3 C. √5•3 D. √5

Answers

Answer:

A. ^3√5 is the answer.

Step-by-step explanation:

5^( 1 / 3 ) = ^3√5

What does 8 represent in the decimal 478.92?​

Answers

Answer:

ones

Step-by-step explanation:

Match each percent with the fraction or decimal equivalent. 5% 0.25 15% 0.07 6.5% 1 20 25% 0.065 7% 3/20​

Answers

Answer:

5% = 1/20

15% = 3/20

6.5% = 0.065

25% = 0.25

7% = 0.07

Step-by-step explanation:

I hope this helps! Just ask me if you have any questions.

Have a nice dayy! :)

Answer:

.05  25%  .15  7% .065  100% 2000% .25

Step-by-step explanation:

Percentages to decimals :

example  

5% = .05

First remove the % sign: 5

Where is the decimal? Here : 5.00

Now move the decimal to the left two places. Fill the gaps with zeros : .05

Another way to do it is by divide the number by 100. Use a calculator if necessary.

Example 2: 6.5%

Move decimal two places to left: .065

Example 3: .56%

Move decimal to places to the left:  .0056

Decimals to percentages. Do the exact opposite. Move decimal to places to the right then add the % sign.

Example: .25 becomes 25%

                .07 becomes 07% or just 7%

x+3/2 + (x-1) = 4/5 ​

Answers

Answer:

x = [tex]\frac{3}{20}[/tex]

Step-by-step explanation:

Given

x + [tex]\frac{3}{2}[/tex] + (x - 1) = [tex]\frac{4}{5}[/tex]

Multiply through by 10 ( the LCM of 2 and 5 ) to clear the fractions

10x + 15 + 10(x - 1) = 8

10x + 15 + 10x - 10 = 8

20x + 5 = 8 ( subtract 5 from both sides )

20x = 3 ( divide both sides by 20 )

x = [tex]\frac{3}{20}[/tex]

x + 3/2 + x - 1 = 4/5

Collect the same terms

x + x + 3/2 - 1 = 4/5

2x + 3/2 - 2/2 = 4/5

2x + (3 - 2)/2 = 4/5

2x + 1/2 = 4/5

2x + 0.5 = 0.8

Subtract both sides 0.5

2x + 0.5 - 0.5 = 0.8 - 0.5

2x + 0 = 0.3

2x = 3/10

Divide both sides by 2

2x ÷ 2 = ( 3/10 ) ÷ 2

x = 3/10 × 1/2

x = 3/20

_________________________________

Check :

3/20 + 3/2 + ( 3/20 - 1 ) = 4/5

3/20 + 30/20 + ( 3/20 - 20/20 ) = 4/5

(3 + 30)/20 + (3 - 20)/20 = 4/5

33/20 - 17/20 = 4/5

(33 - 17)/20 = 4/5

16/20 = 4/5

4 × 4 / 4 × 5 = 4/5

4/5 = 4/5

Thus we found the correct value of x .

And we're done...

Solve the attachment...​

Answers

Answer:

Step-by-step explanation:

=  [tex]\int\limits^1_0 {5x\sqrt{x} } \, dx[/tex]

=  [tex]\int\limits^1_0 {5xx^{1/2} } \, dx[/tex]

= [tex]\int\limits^1_0 {5x^{3/2} } \, dx[/tex]

= 5 [tex]\int\limits^1_0 {x^{3/2} } \, dx[/tex]

= 5*[tex]\frac{2}{5}[/tex]*[tex]x^{5/2}[/tex]  |[tex]\left[\begin{array}{ccc}1\\0\\\end{array}\right] \left[/tex]

= 5*[tex]\frac{2}{5}[/tex]*[tex]1^{5/2}[/tex]

= 2

Answer:

2 ( Option A )

Step-by-step explanation:

The given integral to us is ,

[tex]\longrightarrow \displaystyle \int_0^1 5x \sqrt{x}\ dx [/tex]

Here 5 is a constant so it can come out . So that,

[tex]\longrightarrow \displaystyle I = 5 \int_0^1 x \sqrt{x}\ dx [/tex]

Now we can write √x as ,

[tex]\longrightarrow I = \displaystyle 5 \int_0^1 x . x^{\frac{1}{2}} \ dx [/tex]

Simplify ,

[tex]\longrightarrow I = 5 \displaystyle \int_0^1 x^{\frac{3}{2}}\ dx [/tex]

By Power rule , the integral of x^3/2 wrt x is , 2/5x^5/2 . Therefore ,

[tex]\longrightarrow I = 5 \bigg( \dfrac{2}{5} x^{\frac{5}{2}} \bigg] ^1_0 \bigg) [/tex]

On simplifying we will get ,

[tex]\longrightarrow \underline{\underline{ I = 2 }}[/tex]

f(x)=3x^2 and g(x)=x+2 find (f.g)(x)

Answers

Answer:

3[tex]x^{3}[/tex] + 6[tex]x^{2}[/tex]

Step-by-step explanation:

(f · g )(x) = 3[tex]x^{2}[/tex] · (x + 2) = 3[tex]x^{3}[/tex] + 6[tex]x^{2}[/tex]  (Multiply the two terms by distributing)

Use the given conditions to write an equation for the line in point-slope form and slope-intercept form.
Passing through (-3, -6) and (2,9)
What is the equation of the line in point-slope form?
(Simplify your answer. Use integers or fractions for any numbers in the equation.)
What is the equation of the line in slope-intercept form?

Answers

Answer:

slope-inercept form: y = 3x + 3        point-slope form: y - 9 = 3(x - 2)

Step-by-step explanation:

slope-intercept form:

y2 - y1 / x2 - x1        

9 - (-6)/ 2 - (-3)      

= 3

y = 3x + b

9 = 3(2) + b

9 = 6 + b

3 = b

y = 3x + 3

point-slope form:

y - 9 = 3(x - 2)

On a piece of paper graph f(x) 2x(3)^x then determine which graph represents the function

Answers

Answer:

ok

What do you want me to answer ?

find the gradients of line a and b

Answers

Answer:

Gradient of A: 2

Gradient of B: -1

Step-by-step explanation:

Gradient = change in y/change in x

✔️Gradient of A using two points on line A, (2, 5) and (0, 1):

Gradient = (1 - 5)/(0 - 2) = -4/-2

Simplify

Gradient of A = 2

✔️Gradient of B using two points on line B, (0, 5) and (5, 0):

Gradient = (0 - 5)/(5 - 0) = -5/5

Simplify

Gradient of B = -1

Is it possible for an angle in a triangle to be opposite both the longest side of the triangle and the shortest side of the triangle? Explain.

Answers

Answer: No, it's not possible.

Explanation: An angle must only be opposite exactly one side.

no it’s not possible

Can someone help me solving this problem? I'm confuse so I want help..
question :
a) The length and breadth of a rectangular room are 10m and 8m respectively. How many pieces of carpet 5m long and 2m wide are required to cover it's floor?
Please help me solving this problem !

Answers

Answer: 8

Step-by-step explanation:

Edna earns $680 per week and receives a raise of $95. What is her percentage increase in pay?

Answers

Answer:

13.97%

Step-by-step explanation:

Find her percent increase in pay by dividing 95 by 680:

95/680

= 0.1397

Multiply this by 100:

= 13.97

So, her percent increase in pay is approximately 13.97%

The local skating rink pays Mary a fixed rate per pupil plus a base amount to work as a skating instructor. She earns $90 for instructing 15 students on Monday afternoon. Last Friday, she earned $62 for working with 8 students. Lisa is also a skating instructor. She receives half the base amount that Mary does, but she is paid twice as much per student. Who would earn more money instructing a class of 20 students?
answer choice
1. 120
2. 65
3. 15
4. 10

Answers

Answer:

ok so lets divide 90 by 15 to get 6 so she gets paid 6 per student and that means lisa gets paid 12 so lets just multiply

6*20=120 so mary gets paid 120

and lisa gets paid doable that so 420

but i don't know there base pay so the i can't answer this problem

Hope This Helps!!!

Question
The quotient of a number and 5 has a result of 2. What is the number

Answers

Answer:

Probably 10

Step-by-step explanation:

10/5=2

find the value of the expressions 3x^3-2y^3-6x^2y^2+xy for x=2/3 and y=1/2

Answers

Hi!

[tex]3x^3-2y^3-6x^2y^2+xy=\\\\=3\cdot(\frac{2}{3})^3-2\cdot(\frac{1}{2})^3-6\cdot(\frac{2}{3})^2\cdot(\frac{1}{2})^2+\frac{2}{3}\cdot\frac{1}{2}=\\\\=3\cdot\frac{8}{27}-2\cdot\frac{1}{8}-6\cdot\frac{4}{9}\cdot\frac{1}{4}+\frac{1}{3}=\\\\=\frac{8}{9}-\frac{1}{4}-\frac{2}{3}+\frac{1}{3}=\frac{8}{9}-\frac{1}{4}-\frac{1}{3}=\frac{32}{36}-\frac{9}{36}-\frac{12}{36}=\boxed{\frac{11}{36}}[/tex]

Select the correct answer.

A distribution center is redesigning two of its shipping boxes. The volume of these two boxes needs to be the same, and their dimensions are shown.

Write a system of equations to represent the volume of each box, y. Which statement describes the number and viability of the systems solutions?

Answers

Answer:

The system has two solutions, but only one is viable because the other results in negative side lengths.

Step-by-step explanation:

correct on edmentum or plato

if the slope of one of these perpendicular lines is -4/5 the slope of the other line is ____

Answers

5/4. Perpendicular lines have the opposite value.

HELP QUICKLY PLEASE!! ​

Answers

Answer:

D

Step-by-step explanation:

Because the first equation is equivalent to 1, and d is equivalent to 1, the rest are incorrect

Just Question B!
One of the numbers in the diagram is chosen at random, find the probability that the number is in set B, giving your answer as a decimal.
Gibing brainlist!

Answers

Answer:

0.4 or 0.40

Step-by-step explanation:

There are 10 total numbers. 4 of them are in set B.

So, 4/10 are in set B.

The probability is 0.4 because 4 divided by 10 is 0.4.

Which graph represents the function?
g(x) = {x if x < 2
{-3 if x >= 2

Answers

Step-by-step explanation:We have to graph the function g(x) which is given as:

g(x)=  x when x<2

and -3 when x ≥2.

Clearly after looking at the function we see that the function is not continuous since we find the continuity at x=2 as follows.

Left hand limitat 2:

g(2-h)=lim h→0 2-h

       =2-0=2

Also right hand limit at x=2 is:

g(2+h)=lim h→0 (2+h)

         = 2+0=2

Also g(2)= -3.

As:

Left hand limit= Right hand limit but is not equal to function's value at that point.

Hence, the function is discontinuous at x=2.

so for x<2 we will get a graph of a line y=x.

and for x≥2 we will get a straight line y=-3 parallel to the domain.

For the geometric series 2 + 6 + 18 + 54 + ... , find t n

Answers

Answer:

i believe its 162

Step-by-step explanation:

hope it helped :>

5

Mark is investing $8,000 in an account paying 5.5% interest compounded weekly.
What will Mark's account balance be in 6 years?

Answers

Answer:

you divide a 8000÷ 5.5 yor answer is it

What are equivalent expressions please explain

Answers

Answer:

Equivalent expressions are expressions that are the same, even though they may look a little different. If you plug in the same variable value into equivalent expressions, they will each give you the same value when you simplify.

Step-by-step explanation:

Please help!!!!!!!! I’ve been stuck on this

Answers

5/20

Divide by 5 to simplify will give you 1/4

= 0.25 (in decimal form)

If one angle equals 34º, then the measure of its complement angle is 56º.
true or false

Answers

Answer:

true

Step-by-step explanation:

complementary angles equal 90° and 34+56 is 90°

Answer:

True

Step-by-step explanation:

Complement angle means that the two angles add up to 90 degrees. We can check this by adding the two given angles together:

34+56=90

90=90

This means that this is true.

I hope this helps and have a great day!

the diagram on the right shows a circle.given that the length of arcs RS=2QR , angle QPR=35° and anglePSQ=45°,determine the value of
(a) angle SPR
(b) angle SRP

Answers

9514 1404 393

Answer:

  (a) ∠SPR = 70°

  (b) ∠SRP = 30°

Step-by-step explanation:

The applicable relationships are ...

the measure of an arc is twice the measure of the inscribed angle it subtendsthe sum of measures of the arcs of a circle is 360°the sum of measures of the angles of a triangle is 180°

__

QR is subtended by 35° inscribed angle QPR, so is 2×35° = 70°.

PQ is subtended by 45° inscribed angle PSQ, so is 2×45° = 90°.

RS is 2×QR, so is 2×70° = 140°.

(a) Inscribed angle SPR is half the measure of arc RS, so is 140°/2 = 70°.

__

(b) Arc SP is the remaining arc in the circle, so is ...

  arc SP = 360° -arc PQ -arc QR -arc RS = 360° -90° -70° -140° = 60°.

Inscribed angle SRP is half the measure of arc SP, so is 60°/2 = 30°

find the value of x and HI. H and J are the endpoints​

Answers

Answer:

x = 6

HI = 29

Step-by-step explanation:

✔️HI = ½(AB) => Triangle Mid-segment Theorem

HI = 5x - 1

AB = 58

Plug in the values and solve for x

5x - 1 = ½(58)

5x - 1 = 29

Add 1 to both sides

5x - 1 + 1 = 29 + 1

5x = 30

Divide both sides by 5

5x/5 = 30/5

x = 6

✔️HI = 5x - 1

Plug in the value of x

HI = 5(6) - 1

HI = 30 - 1

HI = 29

Two sides of a triangle have the following measures, Vind the range of possible measures for
the third side,
13) 8,7
14) 12,6
A) 1 B) 2 < x < 14
C) 1 D) 1
A) 8 B) 9 C) 8 < X < 18
D) 6 < X < 18

Answers

Answer:

[tex](13)\ 1 <x < 15[/tex]

[tex](14)\ 6 <x < 18[/tex]

Step-by-step explanation:

Question 13:

[tex]a,b = 8,7[/tex] -- the two sides

Using triangle inequality theorem, we have:

[tex]a + b > x[/tex]

[tex]a + x > b[/tex]

[tex]x + b > a[/tex]

So, we have:

[tex]a + b > x[/tex]

[tex]8 + 7 > x[/tex]

This gives:

[tex]15 > x[/tex]

[tex]a + x > b[/tex]

[tex]8 + x > 7[/tex]

Collect and evaluate like terms

[tex]x > -1[/tex]

[tex]x + b > a[/tex]

[tex]x + 7 > 8[/tex]

Collect and evaluate like terms

[tex]x > 1[/tex]

Ignore the inequality with a negative value.

So, we have:

[tex]x > 1[/tex] and [tex]15 > x[/tex]

Rewrite as:

[tex]1< x[/tex] and [tex]x < 15[/tex]

Merge

[tex]1 <x < 15[/tex]

Question 14:

[tex]a,b = 12,6[/tex] -- the two sides

Using triangle inequality theorem, we have:

[tex]a + b > x[/tex]

[tex]a + x > b[/tex]

[tex]x + b > a[/tex]

So, we have:

[tex]a + b > x[/tex]

[tex]12 + 6 > x[/tex]

This gives:

[tex]18 > x[/tex]

[tex]a + x > b[/tex]

[tex]12 + x > 6[/tex]

Collect and evaluate like terms

[tex]x > -6[/tex]

[tex]x + b > a[/tex]

[tex]x + 6 > 12[/tex]

Collect and evaluate like terms

[tex]x > 6[/tex]

Ignore the inequality with a negative value.

So, we have:

[tex]x > 6[/tex] and [tex]18 > x[/tex]

Rewrite as:

[tex]6< x[/tex] and [tex]x < 18[/tex]

Merge

[tex]6 <x < 18[/tex]

Persons taking a 30-hour review course to prepare for a standardized exam average a score of 620 on that exam. Persons taking a 70-hour review course average a score of 749. Find a linear equation which fits this data, and use this equation to predict an average score for persons taking a 57-hour review course. Round your answer to the tenths place.

Answers

Given:

30-hour review course average a score of 620 on that exam.

70-hour review course average a score of 749.

To find:

The linear equation which fits this data, and use this equation to predict an average score for persons taking a 57-hour review course.

Solution:

Let x be the number of hours of review course and y be the average score on that exam.

30-hour review course average a score of 620 on that exam. So, the linear function passes through the point (30,620).

70-hour review course average a score of 749. So, the linear function passes through the point (70,749).

The linear function passes through the points (30,620) and (70,749). So, the linear equation is:

[tex]y-y_1=\dfrac{y_2-y_1}{x_2-x_1}(x-x_1)[/tex]

[tex]y-620=\dfrac{749-620}{70-30}(x-30)[/tex]

[tex]y-620=\dfrac{129}{40}(x-30)[/tex]

[tex]y-620=\dfrac{129}{40}(x)-\dfrac{129}{40}(30)[/tex]

[tex]y-620=\dfrac{129}{40}(x)-\dfrac{387}{4}[/tex]

Adding 620 on both sides, we get

[tex]y=\dfrac{129}{40}x-\dfrac{387}{4}+620[/tex]

[tex]y=\dfrac{129}{40}x+\dfrac{2480-387}{4}[/tex]

[tex]y=\dfrac{129}{40}x+\dfrac{2093}{4}[/tex]

We need to find the y-value for [tex]x=57[/tex].

[tex]y=\dfrac{129}{40}(57)+\dfrac{2093}{4}[/tex]

[tex]y=183.825+523.25[/tex]

[tex]y=707.075[/tex]

[tex]y\approx 707.1[/tex]

Therefore, the required linear equation for the given situation is [tex]y=\dfrac{129}{40}x+\dfrac{2093}{4}[/tex] and the average score for persons taking a 57-hour review course is 707.1.

Can someone help me with this math homework please!

Answers

Answer:

1/2

8

Step-by-step explanation:

When its talking about the result or output, look at the range, and then follow the line(s) back to the number(s) in the domain. Do the opposite when it's talking about the function of a certain number, e.g. f(4).

Other Questions
Triangle ABC with vertices A(-4, 2), B(4,7), andC(5, 1) in the x-axis.ABCD 13. Water is a polar compound because it exhibits _______charges.O A. negativeO B. positiveO C. oppositeD. neutral A. Tick (1) the best alternatives. 1. What is the acceleration due to gravity on the surface of moon ? (a) 9.8m/s (b)1.6m/s2 (c) 6.67x10-1m/s (d) 9.8m/s? Tone in rhetoric is defined as the:O A. words that make comparisons.O B. way an author feels about a subject.O C. careful choice of words in a piece.O D. sentences that echo one another. great love and great achievement involve great risk nine times a number Carbon disulfide is formed by the reaction of coke (carbon) with sulfur dioxide. How many moles of CS2 will be generated if 8.0 moles of coke react with a surplus of sulfur dioxide? 5C +2502 - CS2 + 4CO A. 0.8 moles B. 1.6 moles C. 3.2 moles OD. 6.4 moles Graph the line.Y=-1/4x+4 ILL MARK BRAINLIEST TO FIRST CORRECT ANSWER Dave solved a quadratic equation. His work is shown below, with Step 111 missing.What could Dave have written as the result from Step 111?\begin{aligned} \dfrac{1}{3}(x+4)^2&=48 \\\\ &&\text{Step }1 \\\\ x+4&=\pm 12&\text{Step }2 \\\\ x=-16&\text{ or }x=8&\text{Step }3 \end{aligned} 31 (x+4) 2x+4x=16=48=12 or x=8Step 1Step 2Step 3Choose 1 answer:Choose 1 answer:(Choice A)A\left(\dfrac{1}{3}x+\dfrac{4}{3}\right)^2=48( 31 x+ 34 ) 2 =48left parenthesis, start fraction, 1, divided by, 3, end fraction, x, plus, start fraction, 4, divided by, 3, end fraction, right parenthesis, squared, equals, 48(Choice B)B\left(\dfrac{1}{3}x\right)^2+\left(\dfrac{4}{3}\right)^2=48( 31 x) 2 +( 34 ) 2 =48left parenthesis, start fraction, 1, divided by, 3, end fraction, x, right parenthesis, squared, plus, left parenthesis, start fraction, 4, divided by, 3, end fraction, right parenthesis, squared, equals, 48(Choice C)C(x+4)^2=144(x+4) 2 =144left parenthesis, x, plus, 4, right parenthesis, squared, equals, 144(Choice D)D(x+4)^2=16(x+4) 2 =16 The population of a large U.S. city is 2,707,210. Which of the following best expresses this population? A 2.7 106 C 2.707 106 B 2.7 107 D 27.07 106 At what rates did she invest?$1500 invested at___%$800 invested at ____% Selected current year company information follows:Net income $15,953Net Sales 712,855Total liabilities, beginning-year 83,932Total liabilities, end-of-year 103,201Total stockholders' equity, beginning-year 198,935Total stockholders' equity, end-of-year 121,851Total asset turnover is:________.a. 2.24 timesb. 2.81 timesc. 3.64 timesd. 4.67 timese. 6.28 times If a(x + 1) + b(x 1) 2 = 0 for all real x, then a =(A) -2(B) -1(C) 0(D) 1(E) 2 Three consecutive integers have a sum of -42. Write and solve an equation to find the three integers. Kh khn ln nht ca min nam trung b v nam b v kh hu l Amy bakes and sells cupcakes out of her home. She uses baking pans that hold 12 cupcakes and 36 cupcakes to fill her orders. Amy needs to fill her biggest order of 504 cupcakes for a special event.a. Write an equation to represent the number of 12-cup and 36-cup baking pans for herorder.b. Amy needs to use a combination of the 12-cup and 36-cup baking pans to fill the order. With only eighteen 12-cup baking pans in her shop, how many of the 36-cup baking pans does she need to complete the order? How is the graph of y=(x-1)2-3 transformed to produce the graph of y-3(x+4)??The graph is translated left 5 units, compressed vertically by a factor of 2, and translated up 3 units.The graph is stretched vertically by a factor of , translated left 5 units, and translated up 3 units.O The graph is translated left 5 units, compressed horizontally by a factor of 3, and translated down 3 units.O The graph is stretched horizontally by a factor of , translated left 5 units, and translated down 3 units. If a physicist performs an experiment, who would likely try to replicate it? The physicist himself Other physicists The family of the physicist No one, because an experiment can never be replicated The Womans Suffrage Movement is responsible for passage of the A. 15th Amendment B. 19th Amendment C. 23rd Amendment D. 26th Amendment how did the lifestyle of the American Indians change with the closing of the American frontier?